LSAT and Law School Admissions Forum

Get expert LSAT preparation and law school admissions advice from PowerScore Test Preparation.

 Jon Denning
PowerScore Staff
  • PowerScore Staff
  • Posts: 904
  • Joined: Apr 11, 2011
|
#9853
Ha, out to lunch? No. Overthinking it? A bit!

Not to mention I think some of the reasoning you use here is a bit off: can the "most profitable" investment available really "decrease in value more than any other investment?" It seems to me those are, by definition, incompatible. That is, the investment that is the most profitable option over a given period of time is simply defined as the one that outperforms the rest (in this case has the highest rate of return). No need to make it any more complicated than that.

Further, don't bother trying to incorporate extra information that isn't supplied in, or defined by, the stimulus. This is a Must be True question, so everything you need is given to you. Here you're given the fairly straightforward idea that when you are losing more than even the best investment possible can gain you, the minimum that you can lose is defined by that best investment. All investments that aren't as profitable will lose more. Hence, losing more means we're dealing with a less profitable investment.

Consider this: at no point will this test ever require, or even expect, you to bring in the level of outside knowledge/consideration that you've attempted here. So as soon as you feel yourself beginning to do that, hit the brakes. Otherwise you're going to get punished for it.

So to answer your question about LSAC expecting you to bring in knowledge: no. They hope you do because they know it will screw you up; they could never require you to in order to answer the question.

And, again, don't ever, EVER question the validity of the information provided to you in a Must be True stimulus. Instead use it to make determinations of consequences and outcomes that said information would support or prove. In this instance your entire knowledge of economics is limited to two sentences; go beyond that and prepare to get burned.

Follow?
 ifstudythenkill
  • Posts: 5
  • Joined: Jun 17, 2013
|
#9854
I follow!

Thank you so much for your help. I'm kind of blown away that Powerscore offers this kind of support!
 Jon Denning
PowerScore Staff
  • PowerScore Staff
  • Posts: 904
  • Joined: Apr 11, 2011
|
#9855
Hey man happy to help! And I'm really stoked that we're able to!

I've always believed that the more interactivity and accessibility that can exist between us and our students (or prospective students), the better both sides are for it.
 LustingFor!L
  • Posts: 80
  • Joined: Aug 27, 2016
|
#38015
I had to simplify the question stem a lot to get through the question stem:

Rate of inflation > rate of return on most profitable investment :arrow: difference between rate is minimum percentage of decline

Particular investment declines by more than that % :arrow: Fill In Blank

Narrowed down to B and C, and incorrectly chose B. Would it be reasonable to remove B as a contender, because it says "becoming" and you cannot prove from the stimulus that it is actively becoming less profitable? The stimulus uses words like will/will be/declines.
 AthenaDalton
PowerScore Staff
  • PowerScore Staff
  • Posts: 296
  • Joined: May 02, 2017
|
#38373
Yes, you're right that we can't prove that the investment is becoming less profitable. It's possible that other factors, such as the inflation rate, are driving the decline.
 Blueballoon5%
  • Posts: 156
  • Joined: Jul 13, 2015
|
#47906
Administrator wrote: Answer Choice (A): This answer is incorrect because the percentage between the most profitable investment available and inflation would have already changed to compensate for this increase.

Answer Choice (D): Just like Answer Choice (A), this answer is incorrect because the percentage between the most profitable investment available and inflation would have already changed to compensate for this decrease.

Answer Choice (E): Once again, this answer is incorrect because the percentage between the most profitable investment available and inflation would have already changed to compensate for this change in which investment happened to be the most profitable.
Hello! I got the right answer and I understand the explanation of the stimulus on this page. However, I am a little confused with the explanation of the answer choices (above). What does it mean when you say, "the percentage between the most profitable investment available and inflation would have already changed to compensate for this increase"?

Here was my line of thinking when answering this question:
1.) I thought answer choice A was wrong because the rate of inflation was constant for both stated investments in the stimulus because the stimulus does not reference any passage of time. One single moment in time = one rate of inflation.
2.) I thought answer choice D was wrong because the stimulus compared two different investments, NOT compared one investment over time.
3.) I thought answer choice E was wrong because there is no indication in the stimulus that the "most profitable available investment" had changed. The stimulus only discusses two investments, and the second investment in premise 2 is clearly not more profitable (since the value percentage declined).

I think I am really confused with the wording of the explanation to these three answer choices. I have never taken a single econ class (and I really don't understand all this econ lingo), so that might be it. Please let me know if you can break this down to novice levels! :-D
 James Finch
PowerScore Staff
  • PowerScore Staff
  • Posts: 943
  • Joined: Sep 06, 2017
|
#62629
Hi Blue Balloon,

To answer your questions:

1) This is what the Administator's original post is trying to explain: we already know what the rate of inflation is, so whether or not it rose to get to that number, or declined, is irrelevant. We only need to know that it is higher than the highest ROI possible and what the difference between the two is.

2) This is the same issue as in answer choice (A); it's irrelevant information, because we only need to know the difference between ROI and inflation, not how they got to the current points. The answer choice doesn't necessarily compare two different investments; the last sentence in the stimulus could (or could not) be referring to the most profitable investment available.

3) Since we don't know whether or not the stimulus is referring to the most profitable investment, and we're not looking at what is happening over time, only at the current moment, this is an irrelevant answer. The original post is trying to explain that we are only looking at the current situation, not what happened in the past.

Hope this clears things up!
 chiickenx
  • Posts: 21
  • Joined: Apr 30, 2019
|
#65595
Hi, I am still very confused with this. I chose the right AC while i took the practice test because while I was caught up between B and C, C seemed like the best since i do not know the implication between value and profits; however, i still do not completely 100% understand this despite reading the entire thread. As the stimulus says, "the difference between those two rate will be the percentage by which, at a minimum, the value of ANY investment will decline."

So, suppose inflation is at 20% and rate of return on the most profitable investment is 15%. If this is the case, the value of ANYinvestment would minimally decline by 5%. The case that we are given is that the value of an investment declined by more than 5% (lets say 7%). From this, the stimulus asks us what must be true.

So my question is that why must it be the case, that the investment whose value declined by 7% cannot be the most profitable investment? It seems to me that the most profitable investment could decline by 7% since the stimulus does not give us a baseline (i.e., we do not know how much the most profitable company's value dropped by, we only know that it must minimally drop by 5%).

From a previous post, it said "When a particular investment is the most profitable one, then it's rate of return, plugged in to the formula they gave us, will always give us the minimum percentage. That is, it's the ruler by which all others are measured. If an investment falls by more than that, it's by definition not the most profitable, and must be less than the most profitable."

However, I do not understand how the most profitable investment is used as a measure... So going back to my hypothetical case, I do not understand why the most profitable investment must decline by only 5%.
 George George
PowerScore Staff
  • PowerScore Staff
  • Posts: 48
  • Joined: Jun 07, 2019
|
#65606
@chiickenx

Everything you wrote using hypothetical percentages (e.g. 5% and 7%) is correct - and shows you were thinking about this problem the correct way. You're also right that the LSAT did not give us a specific "baseline" percentage for the most profitable investment.

But I think what you might've missed is that the second sentence stipulates that the other "particular investment" declines by "more than that percentage." In context, this means that whatever baseline you choose for the most profitable investment, this other investment will have to be declining by a greater percentage. Which brings us back to that quotation you cited. Regardless of what actual percentage the most profitable investment ends up declining by, all others will decline by some larger percentage simply because they're not "the most profitable investment." Hence, "When a particular investment is the most profitable one, then it's rate of return, plugged in to the formula they gave us, will always give us the minimum percentage. That is, it's the ruler by which all others are measured. If an investment falls by more than that, it's by definition not the most profitable, and must be less than the most profitable."

A couple things to note. First, whenever the LSAT uses a superlative, i.e. "most profitable," that tells you there is only one. All others cannot share that attribute, by definition. Second, the comparative language in the second sentence, although ambiguous (as to exactly how much percentage), is still enough information for you to deduce that this other "particular" investment cannot be identical to the "most profitable" one simply because it does not decline by the same percentage as the most profitable investment.
User avatar
 anureet
  • Posts: 22
  • Joined: Aug 06, 2021
|
#91245
I was able to narrow it down to B and C . I just fail to understand the difference between B and C. The last sentence says that, if in such circumstance the value of a particular investment declines by more than that percentage, then isn't it obvious that the investment is becoming less proftable.

For example, say I buy an investment property and the value was decreasing by two percent but now the rate of inflation exceeds the rate of return by 5 percent and now my investment property declines more than that so it's decreasing in value by seven percent. The investment is becoming less profitable because the value is decreasing by seven percent rather than two and I am losing more money than before.

Maybe I understood the stimulus wrong but I fail to see the difference between B and C.

Thank you
Anureet Bhatti

Get the most out of your LSAT Prep Plus subscription.

Analyze and track your performance with our Testing and Analytics Package.